You are on page 1of 3

SOLUZIONI COMPITO DI ALGEBRA 3, 25/01/2013

Esercizio 1. Sia f (X) Q [X] un polinomio irriducibile di grado 3 avente una sola radice
reale R, siano , C R le altre due radici e K = Q (, ) il campo di spezzamento.
i. Mostrare che il gruppo di Galois di K su Q `e isomorfo al gruppo S3 delle permutazioni
di {, , };


ii. Mostrare che Q(), con := ( ) `e lunico sottocampo di K di grado
2 su Q.
Soluzione: Lestensione Q K ha grado n al pi`
u 6. Visto che Q() ha grado 3, essendo f
irriducibile, e che 6 R e quindi 6 Q() ha polinomio minimo f (X)/(X ) Q()[X],
concludo che K ha grado 6 su Q.
Poich`e Q () /Q non `e una estensione normale, GK/Q non `e abeliano. Avendo ordine 6


devo avere GK/Q ' S3 e lisomorfismo si ottiene fissano una biiezione , , {1, 2, 3},
per esempio 1, 2 e 3. I sottogruppi H GK/Q ed i relativi sottocampi sono
dunque i seguenti:
H
GK/Q h(123)i h(23)i h(13)i h(12)i {1}
H
K
Q
Q () Q () Q () Q
 K

KH : Q
1
2
3
3
3
6.
Lunica cosa da verificare `e che Q () = K h(123)i . Osservo che ogni scambio manda 7
h(123)i ma Q() non `
mentre
e contenuto in Q (), Q () o
 (123)() = . Quindi Q() K
Q . La conclusione segue.
Esercizio 2. Sia P (X) Q[X] il polinomio P (X) = X 6 + 5. Sia K un campo di
spezzamento di P (X) su Q.
i. Determinare il grado [K : Q];
ii. Descrivere il gruppo di Galois G di K su Q.
Soluzione: (i) Il polinomio P (X) `e irriducibile su Q per il criterio di Eisentein applicato al
1
` una estensione di grado 6 di Q.
primo 5. Sia = (5) 6 una radice e sia M = Q(). E

j
Le altre radici di P (X) sono della forma 6 con 6 = 1+ 2 3 radice primitiva di ordine 6

dellunit`a in C. Sia L = Q(6 ) = Q( 3). Segue che K `e il campo composto L M . Inoltre


o [K : M ] = 2 e quindi [K : Q] = 12 oppure K = M e [K : Q] = 6.
Il campo L `e una estensione di Galois di Q con
di Galois H isomorfo a Z/2Z e
gruppo
generato dalllautomorfismo

:
L

L
che
manda
3

3.

3
Visto che = 5
M , segue che Q( 5, 3) K. Ma Q( 5, 3) `e il composto
del campo quadratico Q(
15), che `e
contenuto in R,e delcampo quadratico L, che non `e
contenuto in R. Quindi, Q( 15) Q( 3) = Q e Q( 5, 3) ha grado 4 su Q. Concludo
che 4 divide [K : Q] e quindi deve valere [K : Q] = 12.
1

SOLUZIONI COMPITO DI ALGEBRA 3, 25/01/2013

(ii) Segue da (i) che L M = Q e K = Q(6 , ). Sia N = Gal(K/L). Si tratta di un


gruppo ciclico di ordine 6 generato dallautomorfismo : K K che invia 7 6 : so
di avere esattamente 6 automorfismi di K come estensione di L ed ognuno `e univocamente
determinato dallimmagine di e deve inviare in unaltra radice di P (X). Quindi N
= Z/6Z.
= Gal(K/M ) `e isomorfo
Visto che L `e di Galois allora N `e normale in G e il gruppo di Galois H

ad
H tramite
lapplicazione H G G/N = H ed `e generato dallautomorfismo che invia
3 7 3.
Concludiamo che G `e il prodotto semidiretto di N per H. Quindi `e generato da e da .
Visto che (6 ) = 61 ho la relazione
= 1 , che insieme alle relazioni 2 = 1 e 6 = 1
determina G. Concludo che G `e infatti isomorfo al gruppo diedrale di ordine 12.

Esercizio 3. Sia Fp il campo finito con p elementi. Sia F una chiusura algebrica di Fp .
Siano K = Fq ed L = Fq0 estensioni finite di Fp contenute in F con q = pn e q 0 = pm elementi.
i) Determinare condizioni necessarie e sufficienti affinch`e K L.
ii) Nel caso in cui K L dimostrare che lestensione K L `e di Galois con gruppo ciclico.
Soluzione: (i) Per la teoria dei campi finiti sappiamo che per ogni N esiste un unico

sottocampo di F di cardinalit`
a p che coincide con linsieme delle radici del polinomio X p X
in F.
In particolare, K L se e solo se n divide m. Infatti, se n divide m, diciamo m = an,
m
an
(a1)n
n
n
allora ogni elemento K soddisfa p = e quindi p = p = p
= = p =
ovvero L. Viceversa, se K L sia a il grado. Allora pm = |L| = |K|a = pan e quindi
m = an.
n
` un omomorfismo di campi e quindi, per ragioni
(ii) Sia : L L lapplicazione x 7 xp . E
di cardinalit`
a, `e un isomorfismo. Sia H = hi il sottogruppo di Aut(L/K) generato da .
n
Abbiamo LH = {x L|xp = x} = K. Segue per definizione che K L `e di Galois e per il
teorema fondamentale della teoria di Galois H = Gal(L/K).

Esercizio 4. Sia K = Q( 3 + 5).

(1) Si determini un polinomio minimo per 3 + 5.


(2) Si determini linsieme Ram(K) degli ideali primi di Z che ramificano in OK .


` chiaro che K Q[ 3, 5]. Inoltre lunico elemento del gruppo di Galois
Soluzione: E


Gal(Q( 3, 5)/Q) = Gal(Q( 3)/Q) Gal(Q( 5)/Q) ' Z/2Z Z/2Z

che fissa 3 + 5 `e lidentit`


a, per cui K = Q( 3, 5) e il polinomio minimo di 3 + 5 ha
grado 4.
Poich`e

( 3 + 5)2 = 8 + 2 15

3 + 5 soddisfa la relazione
(X 2 8)2 = 60
quindi un suo polinomio minimo `e
X 4 16X 2 + 4

SOLUZIONI COMPITO DI ALGEBRA 3, 25/01/2013

Poniamo K1 = Q( 3) e K2 = Q( 5), allora Disc(K1 ) = 12 mentre Disc(K2 ) = 5: segue


che
"
#
1+ 5
OK = OK1 OK2 = Z
3,
2
Quindi una base intera di K `e

)
1 + 5 3 + 15
1, 3,
,
2
2

Possiamo cos`calcolare

1
3

1 3

Disc(K) = det2
1
3

1
3

1+ 5
2
1+ 5
2
1+ 5
2
1+ 5
2


3+ 15
2
3+ 15

2
3+ 15
2
3+ 15
2

= 24 32 52

Quindi Ram(K) = {2, 3, 5}.

Esercizio 5. Sia {1 , . . . , n } una Q-base del campo di numeri K e supponiamo che


1 , . . . , n OK con d = Disc(1 , . . . , n ). Si dimostri che ogni intero algebrico OK si
scrive nella forma
1 1 + + n n
=
d
con 1 , . . . , n Z.
Soluzione: Siano 1 , . . . , n gli n omomorfismi di K in C e sia K un elemento fissato.
Poich`e {1 , . . . , n } `e una Q-base, esistono x1 , . . . , xn Q tali che
(1)

= 1 x1 + . . . n xn

Applicando i , con i = 1, . . . , n a (1) otteniamo il sistema di n equazioni lineari in n incognite

1 (1 )x1 + + 1 (n )xn = 1 ()

..
..
.
=
.

n (1 )x1 + + n (n )xn = n ()

Poich`e {1 , . . . , n } `e una Q-base, la matrice A = i (j ) `e invertibile e quindi per la regola
di Kramer, xj = j / dove = det(A) e j `e il determinante
della matrice ottenuta da A

sostituendo la colonna j-esima con la colonna i () dei termini noti. Abbiamo che 2 = d
e che , j Z Q = Z; quindi, posto j = dxj in (1), segue la tesi.

You might also like